Are identities used here?

Algebra Level 5

2 0 < i < j < 15 a i a j \large \displaystyle 2 \sum_{0<i<j<15}\sqrt{a_{i}a_{j}}

If a 1 , a 2 , , a 14 a_{1},a_{2},\ldots, a_{14} are positive real numbers such that their sum is 155, find the maximum value of the expression above.


The answer is 2015.

This section requires Javascript.
You are seeing this because something didn't load right. We suggest you, (a) try refreshing the page, (b) enabling javascript if it is disabled on your browser and, finally, (c) loading the non-javascript version of this page . We're sorry about the hassle.

3 solutions

Shivam Jadhav
Nov 17, 2015

By using A . M G . M A.M-G.M we get

ω = 2 ω a 1 a ω ( ( n 1 ) a 1 + ω = 2 ω a ω ) 2 \sum_{ω=2}^{ω}\sqrt{a_{1}a_{ω}}\leq(\frac{(n-1)a_{1}+\sum_{ω=2}^{ω}a_{ω})}{2}

ω = 3 ω a 2 a ω ( ( n 2 ) a 2 + ω = 3 ω a ω ) 2 \sum_{ω=3}^{ω}\sqrt{a_{2}a_{ω}}\leq(\frac{(n-2)a_{2}+\sum_{ω=3}^{ω}a_{ω})}{2} . . . a ω 1 a ω a ω 1 + a ω 2 \sqrt{a_{ω-1}a_{ω}}\leq\frac{a_{ω-1}+a_{ω}}{2}

Adding all above equations we get

2 I < J a I a J ( n 1 ) ( ω = 1 ω a ω ) 2\sum_{I<J}\sqrt{a_{I}a_{J}}\leq(n-1)( \sum_{ω=1}^{ω}a_{ω}) Now putting values we get the answer as 2015 2015 .

Nice solution, but we can streamline it a bit:

0 i < j ( a i a j ) 2 = ( n 1 ) i a i 2 i < j a i a j 0\leq \sum_{i<j} (\sqrt{a_i}-\sqrt{a_j})^2=(n-1)\sum_{i}a_i-2\sum_{i<j}\sqrt{a_ia_j} so 2 i < j a i a j ( n 1 ) i a i = 2015 2\sum_{i<j}\sqrt{a_ia_j}\leq (n-1)\sum_{i}a_i = \boxed{2015}

Otto Bretscher - 5 years, 6 months ago

Log in to reply

Isn't that just Cauchy Schwartz?

Deeparaj Bhat - 5 years, 3 months ago

Log in to reply

Show us how you use Cauchy-Schwarz here!

Otto Bretscher - 5 years, 3 months ago
Mark Hennings
Nov 19, 2015

If a 1 , a 2 , , a n a_1,a_2,\ldots,a_n are positive numbers whose sum is N N , then 2 1 i < j n a i a j = ( i = 1 n a i ) 2 i = 1 n a i = ( i = 1 n a i ) 2 N 2\sum_{1 \le i < j \le n} \sqrt{a_ia_j} \; = \; \left(\sum_{i=1}^n \sqrt{a_i}\right)^2 - \sum_{i=1}^n a_i \; = \; \left(\sum_{i=1}^n \sqrt{a_i}\right)^2 - N Using the Cauchy-Schwarz Inequality, 2 1 i < j n a i a j ( i = 1 n 1 2 ) ( i = 1 n a i ) N = n N N = ( n 1 ) N , 2\sum_{1 \le i < j \le n} \sqrt{a_ia_j} \; \le \; \left(\sum_{i=1}^n 1^2\right)\left(\sum_{i=1}^n a_i\right) - N \; = \; nN - N \; = \; (n-1)N \;, with equality when a 1 = a 2 = = a n = N n a_1=a_2=\ldots=a_n = \tfrac{N}{n} . The maximum value is thus ( n 1 ) N (n-1)N . In this case, where n = 14 , N = 155 n=14,N=155 , the maximum is 2015 2015 .

Harish Sasikumar
Nov 20, 2015

The numbers are all independent. Hence the sum is maximized when each term is maximized. From AM-GM, terms are maximum when all the numbers are equal. That is

a i = a i a_i=a \forall i

Hence 14a=155 There are 91 terms in the sum.

Hence the answer is 2 × 91 × 155 14 = 2015 \frac{2\times 91 \times 155}{14}=2015

0 pending reports

×

Problem Loading...

Note Loading...

Set Loading...